Módulo 2 Med. Interna Flashcards

1
Q

A 25-year-old man comes to the physician because of the rapid onset of pain and swelling of his left knee, which began 24 hours ago. His temperature is 38.5 C (101.3 F), blood pressure is 125/70 mm Hg, pulse is 95/min, and respirations are 20/min. His personal history is significant for IV drug abuse. He denies a history of sexually transmitted diseases. The left knee is tender, swollen, and warm to the touch. Chest auscultation is normal. Which of the following is the most appropriate next step in management?

(A) Blood studies including complete blood count (CBC)
(B) HIV testing prior to instituting any treatment
(C) Plain x-ray films of the joint
(D) Nonsteroidal anti-inflammatory and empiric antibiotic therapy
(E) Diagnostic arthrocentesis

A

Respuesta: E

The correct answer is E. This clinical picture is consistent with septic arthritis. IV drug abusers are prone to developing joint infections (as well as endocarditis) due to Staphylococcus aureus. Fever and local inflammatory changes restricted to a single joint (i.e., monoarthritis) are sufficient clues to the correct diagnosis. Swelling of the joint indicates that there is probably an effusion within the articular cavity. The next step is to confirm the diagnosis and isolate the offending agent by performing aspiration of the joint fluid. Microscopic examination will allow confirming the nature of the effusion (transudate versus exudate) and ruling out crystal- related joint disease (gout and pseudogout). Culture of the fluid will most likely yield S. aureus in this case.

Blood studies, including complete blood count (CBC; choice A), are useful additional investigations. However, arthrocentesis is more important in management.

HIV testing (choice B) is appropriate in this case, considering the high frequency of HIV infection among IV drug abusers. However, the results would not have any influence on the specific therapy for septic arthritis.

Plain x-ray films of the joint (choice C) are not helpful in diagnosis or management of infectious arthritis.

Nonsteroidal anti-inflammatory and empiric antibiotic therapy
(choice D) prior to arthrocentesis would be a mistake in this context, since isolation of the pathogen is necessary to institute appropriate antibiotic therapy.

How well did you know this?
1
Not at all
2
3
4
5
Perfectly
2
Q

A 28-year-old man presents to the emergency department with complaints of fever, chills, and malaise for the past 3 days. He also complains of nausea, headaches, and anorexia. The patient denies any homosexual practices but admits to occasional IV drug use. Examination of his palms and soles reveals painless macules; on auscultation, a loud holosystolic murmur is noted. Which of the following is the most appropriate next step in diagnosis?
(A) ECG
(B) Echocardiogram
(C) Chest CT scan with contrast
(D) RPR for syphilis
(E) Cardiac catheterization

A

Respuesta: B

The correct answer is B. This patient is displaying signs and symptoms of acute bacterial endocarditis (ABE), with fever, chills, a heart murmur, Janeway lesions, and a positive history of drug use. A thoracic echocardiogram is the most appropriate first step for finding the vegetations of ABE on heart valves, which are diagnostic. Blood cultures are also diagnostic, but take 2 days for a result and confirm only the bacteremia itself, not its source. Since the mortality is great for ABE, empiric antibiotics should be initiated after three sets of blood cultures are drawn. The organism is most likely Staphylococcus aureus, and the right-sided valves are more commonly affected in IV drug users. The tricuspid regurgitation murmur (a holosystolic murmur along the sternal border that increases with inspiration) should always suggest the diagnosis of S. aureus endocarditis.

ECG findings (choice A) are not sensitive for diagnosing endocarditis.

There is no role for CT (choice C) or cardiac catheterization (choice E) in this case. The findings do not suggest syphilis; however, both an HIV test and an RPR (choice D) would provide useful information, although neither would aid in the diagnosis of ABE.

How well did you know this?
1
Not at all
2
3
4
5
Perfectly
3
Q

A 71-year-old man is brought to the emergency department with acute onset of headache, vomiting, and confusion. The family reports that he has a long history of poorly controlled hypertension with hypertensive renal disease and eye disease that were diagnosed 3 years ago. They report that, a few hours ago, he rapidly developed a very severe headache, and over the next half hour, became more lethargic and confused, and had five episodes of vomiting. His blood pressure is 235/140 mm Hg in both arms, and he appears to have a lateral gaze paralysis on the right. There is no nuchal rigidity, and the pupils appear reactive bilaterally; however, papilledema is evident on funduscopic exam. Which of the following is the most likely diagnosis?
(A) Cerebellar hemorrhage
(B) Epidural hematoma
(C) Putamenal hemorrhage
(D) Subarachnoid hemorrhage
(E) Subdural hematoma

A

Respuesta: C

The correct answer is C. This patient is having a hypertensive hemorrhage. The caudate and the putamen are the most common sites for such bleeds (70%), which can lead to dangerous elevations in intracranial pressure (ICP), as in this patient. The signs and symptoms of increased ICP, when present, portend imminent herniation of the brain and certain death. These patients require urgent intervention to lower their blood pressure.

The cerebellum (choice A) is an uncommon site (< 5%) for
hypertensive hemorrhage. When cerebellar hemorrhages occur,
urgent intervention is required because they can cause brainstem compression and/or obstructive hydrocephalus.

Epidural hematoma (choice B) is usually the result of trauma to the squamous portion of the temporal bone of the skull and is not associated with hypertension.

Subarachnoid hemorrhage (SAH; choice D) is infrequently
associated with severe hypertension and is usually accompanied by meningismus. Once the SAH is identified, neurosurgical
intervention to stop the bleeding can be performed, and the patient thereafter has a normal life expectancy. The most common nontraumatic cause for SAH is a berry aneurysm in the anterior portion of the circle of Willis.

Subdural hematoma (choice E) results from tearing of the bridging subdural veins, most often due to trauma or shearing forces. It is uncommon without trauma and, even when present, does not tend to produce headache and increased ICP unless very severe.

How well did you know this?
1
Not at all
2
3
4
5
Perfectly
4
Q

A 55-year-old woman presents to the emergency department because of chest pain. The pain, which has lasted 3 hours, is substernal and dull in nature, with no relation to respiration or position. The pain does not radiate and is accompanied by weakness, lightheadedness, and nausea. She has received oxygen, aspirin, a continuous infusion of nitroglycerin, and a beta blocker. Her chest x-ray film is normal, and her ECG is remarkable for inverted T waves in leads II, III, and aVF. Which of the following is the most important next step in management?
(A) Nifedipine
(B) IV heparin
(C) IV thrombolytic therapy
(D) Cardiac catheterization
(E) Percutaneous coronary angioplasty

A

Respuesta: B

The correct answer is B. The patient has unstable angina. Unstable angina with ECG changes is associated with critical coronary artery stenosis in most cases. One goal of therapy is to prevent thrombus formation on complex atherosclerotic plaques; heparin is the most effective proven treatment to prevent progression of unstable angina to myocardial infarction. Heparin is also required to maintain vessel patency when using relatively fibrin-specific thrombolytics, such as tPA. Heparin may cause delayed thrombocytopenia in about 10% of cases.

Nifedipine (choice A), a calcium-channel blocker, has no proven benefit in the therapy of acute myocardial infarction. The drug decreases afterload and may cause a reflex tachycardia.

The patient does not meet the criteria for thrombolytic therapy (choice C). The best candidate for thrombolytic therapy is one in whom the ECG has distinct regional ST segment elevation or new left bundle branch block. Thrombolytic therapy has not been shown to benefit patients with inverted T waves, ST segment depression, or nonspecific ST-T waves changes and chest pain.

The patient may eventually require catheterization (choice D) to see the extent of the coronary artery disease, but it is not the appropriate next step in management.

Angioplasty (choice E) should be considered if chest pain refractory to medical management persists, but it is not the appropriate next step in management.

How well did you know this?
1
Not at all
2
3
4
5
Perfectly
5
Q

A 40-year-old man consults a physician because of dizziness. The patient has noticed that every time he lays with the right side of his head down, he develops a whirling sensation within a few seconds. This symptom will last as long as the position is maintained, but resolves when a new head position is taken. He does not experience tinnitis or hearing changes during these episodes. Otoscopic examination is within normal limits. Which of the following is the most likely diagnosis?

(A) Benign paroxysmal positional vertigo
(B) Cholesteatoma
(C) Herpes zoster oticus
(D) Meniere disease
(E) Presbyacusis

A

Respuesta: A

The correct answer is A. This patient has benign paroxysmal positional vertigo. The pathophysiology appears to involve granular masses (tiny rocks) that sit on the cupola in the inner ear, pushing the cilia (hairs) on the sensory cells down. Certain positions compress the cells more, producing vertigo. Patients should be instructed to avoid the position that sets off the vertigo. A canalith repositioning maneuver is effective in most cases, but chronic cases may require surgical treatment. Some cases resolve spontaneously within a year.

Cholesteatoma (choice B) is a tumor-like, benign lesion that can destroy the middle ear and occurs in the setting of chronic otitis media.

Herpes zoster oticus (choice C), or herpes infection of the ganglion of CN VIII, causes severe ear pain, vertigo, hearing loss, and sometimes facial nerve paralysis.

Meniere disease (choice D) causes the cluster of vertigo, tinnitis, and fluctuating hearing loss, but is usually not triggered by positional changes.

Presbyacusis (choice E) is a progressive loss of sensitivity to high frequencies with age.

How well did you know this?
1
Not at all
2
3
4
5
Perfectly
6
Q

A 62-year-old woman with a history of depression and hypertension presents complaining of recurrent falls over the past 6 months. She had been having difficulty with complex tasks at work and was recently asked to leave. On examination, her mental status is unremarkable. Her cranial nerve examination is notable for limited downward gaze. She has prominent, symmetric bradykinesia with more axial than limb rigidity. There is no resting tremor. Her gait is stiff with “en bloc” turning. Reflexes are normal, with downgoing toes. An MRI of the brain is read as showing a small lacunar infarct in the left putamen. She has recently been started on L- dopa/carbidopa but has had little to no improvement in her symptoms. Which of the following is the most likely diagnosis?

(A) Basal ganglia stroke
(B) Carbon monoxide poisoning
(C) Cervical stenosis
(D) Parkinson disease
(E) Progressive supranuclear palsy

A

Respuesta: E

The correct answer is E. Progressive supranuclear palsy is a degenerative disorder that predominantly affects the midbrain and basal ganglia. The clinical hallmarks are symmetric parkinsonism with vertical gaze limitation and axial rigidity. These patients tend to have falls as their earliest symptoms. There is an associated mild-to-moderate dementia that usually involves frontal lobe functions more than hippocampal/memory systems. These patients tend to show a very modest response, if any, to L-dopa/carbidopa. Imaging is typically unremarkable.

The basal ganglia (choice A) is a common site of small lacunar strokes in patients with hypertension and/or diabetes. Most of these tend to be asymptomatic, and vascular parkinsonism requires a heavier burden of disease in the basal ganglia. In addition, a unilateral left putamenal lacunar infarct would result in right-sided, rather than symmetric, symptoms.

Carbon monoxide poisoning (choice B) results in bilateral pallidal (globus pallidus) necrosis. These patients develop symmetric parkinsonian symptoms but would not be expected to have vertical gaze problems. In addition, an MRI would demonstrate bilateral lesions in the putamen.

Cervical stenosis (choice C) with impingement on the spinal cord can present with falls and a spastic gait but should not affect eye movements or cognition. These patients should have hyperreflexia and upgoing toes.

Idiopathic Parkinson disease (choice D) typically begins asymmetrically, with resting tremor and rigidity worse on one side of the body. Vertical eye movements are not usually affected. Falls occur but normally a few years into the course of the disease. Patients with idiopathic Parkinson disease will usually experience a pronounced improvement in symptoms when started on Sinemet.

How well did you know this?
1
Not at all
2
3
4
5
Perfectly
7
Q

A 22-year-old man presents with burning on urination and a milky urethral discharge for 3 days. He had unprotected sex 5 days prior to the onset of these manifestations. A smear of the urethral discharge demonstrates gram-negative diplococci in neutrophilic granulocytes. The patient reports no allergies. Which of the following is the most appropriate treatment?

(A) Amoxicillin
(B) Azithromycin
(C) Ceftriaxone
(D) Doxycycline
(E) Penicillin G

A

Respuesta: C

The correct answer is C. This is the typical presentation of gonorrhea in men. The infection may regress spontaneously, progress to involve the epididymis and prostate, or become chronic, resulting in urethral strictures. In women, the infection is more often clinically silent, but when symptomatic, the manifestations frequently begin during menses, with frequency, dysuria, and urethral discharge. Chronic cervicitis is an important reservoir of gonococci. If gonococci cannot be demonstrated in smears of the discharge, cultures become necessary. For uncomplicated urethritis or cervicitis, a single intramuscular injection of ceftriaxone, 125 mg, is the treatment of choice and guarantees compliance.

Amoxicillin (choice A) and penicillin G (choice E) are no longer recommended because of the increasing prevalence of penicillin-resistant strains of gonococcus.

Chlamydial infection develops frequently in association with gonorrhea. Thus, therapy with ceftriaxone should be combined with a drug effective against chlamydia. Erythromycin, 500 mg 4 times daily for 1 week, or alternatively azithromycin (choice B) in a single oral dose of 1 g, may be used.

Doxycycline (choice D) is also effective against chlamydia and should be administered at a dosage of 100 mg twice daily for 1 week.

How well did you know this?
1
Not at all
2
3
4
5
Perfectly
8
Q

A 50-year-old woman presents with a chief complaint of dizziness when she gets out of bed in the morning. During the episodes of dizziness, she feels very warm and flushed. She admits to frequent episodes of abdominal cramping and severe watery diarrhea during the past year, and she recently began having dry, itchy skin. On physical examination, she has a 2/6 systolic murmur best heard at the left lower sternal border. No wheezing, rhonchi, or crackles are apparent on lung examination. Laboratory results are unremarkable. Which of the following is the most appropriate next step in diagnosis?

(A) Barium examination of the bowel
(B) Measurement of serum lipase and amylase levels
(C) Ultrasound of the abdomen
(D) Urinalysis for 5-hydroxyindoleacetic acid (5-HIAA)
(E) Small bowel biopsy

A

Respuesta: D

The correct answer is D. This woman has carcinoid syndrome. The classic triad of this disorder is flushing (present in 85%), watery diarrhea, and valvular heart disease. The first test for screening carcinoid syndrome is the determination of 5-HIAA (metabolite of serotonin, 5-HT) in a 24-hour urine sample (carcinoid patients may excrete more than 25 mg/day). Carcinoid syndrome is also associated with hypotension, bronchospasm, telangiectasia, and abdominal cramps due to the release of serotonin and vasoactive peptides, especially in bronchial carcinoid. There may also be a secondary niacin deficiency, causing dermatitis, depression, and diarrhea. Symptomatic treatment of carcinoid syndrome consists of giving the synthetic peptide octreotide.

Barium examination of the bowel (choice A) will often not
demonstrate the primary carcinoid tumor, most commonly located in the distal ileum.

Measurement of serum lipase and amylase levels (choice B) is indicated in the diagnosis of pancreatitis.

Ultrasound examination (choice C) would be indicated if she had symptoms pointing to an abdominal mass or gallbladder disease.

Small bowel biopsy (choice E) is invasive and would not aid in the diagnosis.

How well did you know this?
1
Not at all
2
3
4
5
Perfectly
9
Q

A 40-year-old woman consults a physician about a “mole” on her neck. The lesion is 2 cm in diameter and slightly irregular, and has a variegated dark red to brown to black color. Wide excision of the lesion demonstrates a malignant neoplasm that extends to a depth of 3 mm. This tumor would be most likely to stain for which of the following tumor markers?

(A) Alpha-fetoprotein
(B) CA-125
(C) Leukocyte common antigen (LCA)
(D) Prostate specific antigen (PSA)
(E) S-100

A

Respuesta: E

The correct answer is E. The tumor is a malignant melanoma, which is a neoplastic proliferation of melanocytes. The cells in these tumors are related to neuroendocrine cells and often stain immunohistochemically for S-100. The prognosis in malignant melanoma is closely related to the depth of the lesion, since shallow lesions are much less likely to metastasize than are lesions of 1 mm or more thickness, which have reached the rich lymphatic plexus of the superficial dermis. The result is that shallow melanomas have a close to 100% cure rate with wide (typically 1 cm) excision, whereas deep melanomas have a dreadful prognosis since they typically have already metastasized by the time of surgical removal and usually fail to respond to chemotherapy.

Alpha-fetoprotein (choice A) is a marker for testicular and ovarian tumors with a yolk sac component, as well as hepatocellular carcinoma.

CA-125 (choice B) is a marker for some ovarian tumors.

LCA (choice C) is a marker for some lymphoid neoplasms.

PSA (choice D) is a marker for prostatic carcinoma.

How well did you know this?
1
Not at all
2
3
4
5
Perfectly
10
Q

A 19-year-old homosexual college freshman presents to the student health clinic complaining of diarrhea of 2 days’ duration. He returned 9 days ago from a spring break trip to Mexico. His diarrhea is accompanied by prominent bloating, flatulence, nausea, and general malaise. On physical examination, he appears to be well hydrated, with a temperature of 36.9 C (98.4 F), blood pressure of 130/86 mm Hg, pulse of 89/min, and respirations of 18/min. Examination of the abdomen reveals diffusely hyperactive bowel sounds without tenderness or masses. A stool sample is negative for red and white blood cells. Which of the following is the most appropriate next step in management?

(A) Supportive care with IV fluids
(B) Treatment with ciprofloxacin
(C) Treatment with mebendazole
(D) Treatment with metronidazole
(E) Treatment with trimethoprim-sulfamethoxazole

A

Respuesta: D

The correct answer is D. This patient probably has giardiasis, caused by Giardia lamblia. The diagnosis of giardiasis can often be made clinically on the basis of symptoms of flatulence and bloating appearing several days after a trip to Mexico (and endemic zones). Metronidazole is the treatment of choice for giardiasis.

Supportive care with IV fluids (choice A) is used for patients with Escherichia coli “traveler’s diarrhea” who are severely dehydrated. Patients generally have diarrhea the day after they return from their trip, rather than 1 week later.

Ciprofloxacin (choice B) is effective for Shigella and Salmonella infections. Patients usually have fever and blood or leukocytes in the stool, since these agents are invasive.

Mebendazole (choice C) is indicated for infection with helminths.

Trimethoprim-sulfamethoxazole (choice E) is not effective against Giardia lamblia.

How well did you know this?
1
Not at all
2
3
4
5
Perfectly
11
Q

A 26-year-old woman has a 10-year history of type 1 diabetes mellitus. She has maintained strict glycemic control and has had no significant diabetic complications so far. On her last routine examination, her blood pressure is 125/78 mm Hg. Blood chemistry studies are within normal limits. Funduscopic examination reveals no evidence of diabetic retinopathy. Which of the following is the most appropriate next step in management to prevent diabetic nephropathy?

(A) Periodic measurement of serum creatinine levels
(B) Screening for microalbuminuria with dipstick examination of urine
(C) Screening for microalbuminuria with 24-hour urine collection
(D) Administration of ACE inhibitors
(E) Renal biopsy

A

Respuesta: D

The correct answer is C. Diabetes mellitus is the most common cause of chronic renal failure in the U.S. (and probably in all industrialized countries). Diabetic nephropathy is one of the most severe complications and manifests on average 10-15 years after the onset of diabetes. The earliest expression of diabetic nephropathy is microalbuminuria, while the patient is otherwise asymptomatic. This is the rationale for screening diabetic patients for microalbuminuria, which should be performed by 24-hour urine collection or on an early morning urine sample. In the latter case, dipstick screening (choice B) may not be sufficiently sensitive. The albumin:creatinine ratio in an early morning urine sample is a convenient alternative to 24-hour collection. A ratio < 3.5 is normal and >10 is abnormal; between these two values, reevaluation is recommended. During the phase of microalbuminuria, aggressive treatment, including strict glycemic and blood pressure control, is in order.

Measurement of serum creatinine levels (choice A) would not be valuable in detecting preclinical renal damage.

Treatment with ACE inhibitors (choice D) has been shown to slow progression of renal nephropathy, possibly because of the reduction of intraglomerular pressure. This treatment is not widely used if microalbuminuria absent and the patient is normotensive.

Renal biopsy (choice E) is not indicated in asymptomatic diabetic patients as a method for prevention of renal disease.

How well did you know this?
1
Not at all
2
3
4
5
Perfectly
12
Q

A 45-year-old woman consults a physician because of chronic fatigue. A review of systems reveals long-standing stomach problems characterized by slow digestion and delayed emptying of her stomach. A complete blood count demonstrates a moderately severe megaloblastic anemia. Serum vitamin B12 level is 85 pg/mL; serum folate is 3 ng/mL; and serum iron is 105 mg/dL. Autoantibodies to intrinsic factor are detected in the serum. A biopsy of the stomach is most likely to show which of the following?

(A) Acute erosive gastritis
(B) Gastric atrophy
(C) Linitis plastica
(D) Menetrier disease
(E) Peptic ulcer

A

Respuesta: B

The correct answer is B. The patient has pernicious anemia, in which gastric atrophy is associated with megaloblastic anemia due to vitamin B12 deficiency. The gastric atrophy characteristically involves the corpus, with sparing of the antrum. Most cases appear to have an autoimmune basis, with antibodies to parietal cells detected in 90% of patients; antibodies to intrinsic factor and the proton pump (H+/K+ ATPase) are also commonly present. The lack of parietal cells and the damage to the proton pump lead to markedly decreased acid secretion by the stomach. Lack of intrinsic factor leaves the small bowel unable to absorb vitamin B12, leading to megaloblastic anemia.

Acute erosive gastritis (choice A) is seen most often in severely ill patients, who develop multiple small gastric ulcers.

Linitis plastica (choice C) is an aggressive form of adenocarcinoma of the stomach that produces a “leather bottle” stomach.

Menetrier disease (choice D) is characterized by markedly
thickened gastric folds with mucous gland hyperplasia. It presents with weight loss and severe protein wasting because of protein loss from the gastric mucosa.

Peptic ulcer disease (choice E) usually occurs in the setting of Helicobacter pylori infection or NSAID use and is very rare in pernicious anemia, since gastric acid secretion is markedly diminished.

How well did you know this?
1
Not at all
2
3
4
5
Perfectly
13
Q

A 72-year-old man who was recently diagnosed with lymphoma has been undergoing chemotherapy for the past 3 weeks. He now develops acute renal failure. His laboratory studies reveal a creatinine of 4 mg/dL, urea nitrogen of 15 mg/dL, and uric acid level of 20 mg/dL. Which of the following would most likely have prevented this patient’s acute renal failure?

(A) Allopurinol
(B) Diphenhydramine
(C) Furosemide
(D) N-acetylcysteine
(E) Nifedipine
(F) Urinary acidification

A

Respuesta: A

The correct answer is A. Instituting chemotherapy in this patient has resulted in tumor lysis syndrome (TLS). TLS results from the the acute lysis of lymphoma cells and the acute renal failure from the precipitation of uric acid and hypoxanthine in the renal collecting tubules. Patients should receive allopurinol, a xanthine oxidase inhibitor that reduces the synthesis of uric acid, and should be aggressively hydrated prior to the initiation of chemotherapy to reduce the incidence of TLS.

Diphenhydramine (choice B) is an antihistamine that may be used in allergic conditions. Because TLS is not due to a drug allergy, diphenhydramine has no role in its prevention or management.

Furosemide diuretic (choice C) is reserved for well hydrated patients with insufficient diuresis. It increases the excretion of water but has not been proven to be beneficial as front-line therapy in TLS. It may contribute to uric acid or calcium phosphate precipitation in renal tubules in volume-contracted patients.

N-acetylcysteine (choice D) is used in the treatment of acetaminophen overdose and may be used to reduce hemorrhagic cystitis due to cyclophosphamide and ifosfam. However, it has no role in the prevention of TLS.

Nifedipine (choice E) is a calcium channel blocker used to treat hypertension and angina. It has no role in the prevention of TLS.

Urinary alkalinization, not urinary acidification (choice F), is a method of managing TLS. Intravenous sodium bicarbonate promotes alkaline diuresis and acts to solubilize (and thus minimize) intratubular precipitation of uric acid.

How well did you know this?
1
Not at all
2
3
4
5
Perfectly
14
Q

A 30-year-old man consults a physician because of weight loss and fatigue. A complete blood count demonstrates an erythrocyte count of 2.2 million/mm3, a leukocyte count of 105,000/mm3, and a platelet count of 100,000/mm3 . The peripheral smear shows many abnormal white cells containing multiple Auer rods. Remission is achieved with chemotherapy, and the decision is made to treat the patient with total body irradiation followed by allogeneic bone marrow transplantation. Depletion of which of the following cells in the transplanted marrow tends to decrease the incidence of subsequent graft-versus-host disease?

(A) B cells
(B) Megakaryocytes
(C) Promyelocytes
(D) Pronormoblasts
(E) T cells

A

Respuesta: E

The correct answer is E. The patient has acute myeloid leukemia (AML). Auer rods are pathognomonic for AML. Patients who have this disease and undergo bone marrow transplantation in the first remission have a 50 to 60% chance of long-term, disease-free survival. The major complications of allogeneic bone marrow transplantation are infection, failure of graft survival, recurrent leukemia, and graft-versus-host disease. The incidence of graft-versus-host disease can be reduced by removal of T cells from the donor marrow by using monoclonal antibodies, rosetting techniques, or mechanical separation.

B cells (choice A) are not as important as T cells in graft-versus- host disease.

Megakaryocytes (choice B), promyelocytes (choice C), which are granulocyte precursors, and pronormoblasts (choice D), which are erythrocyte precursors, play no role in graft-versus-host disease.

How well did you know this?
1
Not at all
2
3
4
5
Perfectly
15
Q

A 40-year-old woman presents to her physician’s office with a rash on her legs for the past 4 days. She recalls a recent respiratory infection. On examination, there is a small amount of blood in her nostrils. There are several hemorrhagic bullae in her oral cavity. Her lungs are clear, cardiac examination is unremarkable, and her abdomen is soft with no palpable spleen or liver. Both lower extremities have multiple dark blue ecchymoses.

Laboratory analysis reveals:
Leukocyte count: 9000/mm3
Hemoglobin: 10.1 g/dL
Platelets: 9000/mm3
Peripheral smear: Reticulocytosis with normal erythrocytes and megathrombocytes

An ultrasound examination is negative for masses or fluid collections. Which of the following is the most appropriate next step in management?

(A) Cryoprecipitate
(B) Immunoglobulins
(C) Prednisone
(D) Plasmapheresis
(E) Splenectomy

A

Respuesta: C

The correct answer is C. This woman most likely has idiopathic thrombocytopenic purpura (ITP). ITP is most common in adults (women > men) aged 20-40 years. Bleeding, epistaxis, oral bleeding, or menorrhagia can occur, and isolated thrombocytopenia (< 10,000) is characteristic. Ten percent of patients will have coexistent autoimmune hemolytic anemia (note the reticulocytosis and anemia). The first-line therapy is prednisone if the patient is not actively bleeding. Patients who are bleeding may require IV immunoglobulin (choice B) to block phagocytic activity; in severe cases, splenectomy (choice E) may be required.

Cryoprecipitate (choice A) is effective treatment for von Willebrand disease.

Plasmapheresis (choice D) is the treatment of choice for hemolytic uremic syndrome (HUS) in a coagulation disorder setting. The most striking features of HUS are fever, fragmented RBC, and renal failure without neurologic signs. HUS is often seen after a diarrheal illness, particularly after infection with Escherichia coli 0157:H7.

How well did you know this?
1
Not at all
2
3
4
5
Perfectly
16
Q

A 30-year-old man presents with a rapidly enlarging, single, stony hard, palpable 2.5-cm nodule in his thyroid gland. Thyroid isotope scanning demonstrates the nodule to be “cold.” On resection of the thyroid gland with subsequent pathologic examination, the nodule is found to contain follicular structures, some of which have inwardly protruding fibrovascular branching cores covered by epithelial cells. Many of the epithelial cells have “orphan Annie” nuclei. Which of the following is the most likely diagnosis?

(A) Follicular carcinoma of the thyroid
(B) Graves disease
(C) Hashimoto disease
(D) Nontoxic goiter
(E) Papillary carcinoma of the thyroid

A

Respuesta: E

The correct answer is E. Papillary structures within follicles that have epithelial cells with nuclei with cleared centers (“orphan Annie eyes”) indicate the presence of papillary carcinoma of the thyroid. It does not matter whether the papillary structures are present in only a percentage of the follicles; the condition is still considered to be papillary carcinoma. Of all thyroid cancers, 60% to 70% are papillary carcinomas. The condition is more frequent in younger patients but tends to be more aggressive in the elderly. There is usually a single dominant nodule that is “cold” (does not take up radioactive iodine) on thyroid scan. Among the different types of thyroid cancers, papillary carcinoma tends to be the one with the best prognosis overall, and smaller lesions can be treated with thyroid lobectomy alone. Large or more diffusely spreading lesions require complete thyroidectomy, sometimes with ablation of any residual thyroid tissue with large doses of 131I.

Follicular carcinoma of the thyroid (choice A) would not exhibit the papillary structures or orphan Annie nuclei seen in this case. Well- differentiated follicular carcinoma can be very difficult to distinguish from normal thyroid tissue.

Graves disease (choice B) would be characterized by prominent hyperthyroid symptoms and would show smaller than normal amounts of colloid on biopsy. This disease is not characterized by a single hard nodule; a symmetric, diffusely enlarged goiter may be found.

Hashimoto disease (choice C) would produce diffuse goiter and would show an intense lymphocytic infiltrate with destruction of follicles on biopsy. Symptoms of hypothyroidism are often apparent.

Nontoxic goiter (choice D) produces a goiter that may be either smooth or multinodular, but does not usually have a single dominant nodule. On biopsy, the follicles are typically of a wide range of sizes, and the patient is usually clinically euthyroid.

How well did you know this?
1
Not at all
2
3
4
5
Perfectly
17
Q

A 38-year-old man is admitted to the hospital after sustaining a pulmonary embolism. The patient has a past medical history significant for two idiopathic deep venous thromboses and takes only an aspirin daily. Three hours ago, he developed acute shortness of breath, pleuritic chest pain, and palpitations. He was taken by ambulance to the hospital. In the emergency department, he was diagnosed with a pulmonary embolus on the basis of clinical signs and symptoms and a ventilation-perfusion scan. He was started on IV unfractionated heparin at that time. Which of the following laboratory tests would be most appropriate to guide therapy with this drug?

(A) Bleeding time
(B) Factor Xa levels
(C) Platelet count
(D) Prothrombin time (PT)
(E) Partial thromboplastin time (PTT)

A

Respuesta: E

The correct answer is E. The dose of traditional unfractionated heparin required for anticoagulation can be determined by following the partial thromboplastin time (PTT). Heparin prolongs the PTT. This test is performed by adding particulate matter to a patient’s blood sample to activate the intrinsic coagulation cascade; the PTT therefore reflects activity of the intrinsic coagulation pathways.

Bleeding time (choice A) reflects the interaction of platelets with the vascular endothelium leading to the formation of an initial clot. An abnormal bleeding time usually reflects abnormal or diminished platelets.

Factor Xa levels (choice B) are used to follow the dosing of the newer, low-molecular-weight heparins.

Platelet count (choice C) may be followed while giving IV heparin, since a significant minority of patients will develop heparin-induced thrombocytopenia. The platelet count, however, is not the test used to monitor efficacy of heparin therapy and any dosing changes.

Prothrombin time (PT; choice D) is a measure of the extrinsic coagulation system. This value, and the corresponding international normalized ratio (INR) of patient and normal PTs, is particularly sensitive to deficiencies in factor VII. It is usually used to help guide Warfarin therapy.

How well did you know this?
1
Not at all
2
3
4
5
Perfectly
18
Q

A 48-year-old man presents to the physician’s office with progressive hearing loss in his right ear for the past several months. He describes a ringing and hissing sound in his right ear, and he feels unsteady on his feet, as if he is losing his sense of balance. His past medical history includes syphilis, which was treated at age 20, and bronchial asthma, which is controlled with medications. On physical examination, right-sided facial numbness is noted, and a Rinne test shows air conduction that is greater than bone conduction. Routine laboratory profile is normal, and rapid plasma reagin is negative. Which of the following is the most likely diagnosis?

(A) Acoustic neuroma
(B) Benign positional vertigo
(C) Lyme disease
(D) Ménière disease
(E) Tertiary syphilis

A

Respuesta: A

The correct answer is A. Acoustic neuroma, also known as neurilemoma or schwannoma, is a benign tumor that typically arises from the neurilemmal sheath of the vestibular portion of the acoustic nerve in the auditory canal. Symptoms are produced by compression or displacement of the cranial nerves, brainstem, and cerebellum and by obstruction of CSF flow. The trigeminal (CN V) and facial (CN VII) nerves are often affected because of their anatomic location and relationship to the acoustic nerve. Clinical findings include insidiousonset of sensorineural hearing loss, tinnitus, and a sensation of fullness in the ear. Facial numbness, facial weakness, headache, and gait ataxia may also be present; vertigo ultimately develops in 20% to 30% of patients. The most useful diagnostic test is MRI of the cerebellopontine angle. Treatment is surgical excision of the lesion.

Patients with benign positional vertigo (choice B) experience vertiginous symptoms only when the head is in a specific position. Symptoms are usually most severe when the patient is in the lateral decubitus position with the affected ear down. Hearing loss is not a feature of this condition.

There is no specific reason at this time to suspect Lyme disease (choice C), although it should be included in the differential.

Ménière disease (choice D) is characterized by repeated episodes of vertigo lasting minutes to days, tinnitus, and progressive sensorineural hearing loss.

Tertiary syphilis (choice E) or neurosyphilis presents 3 to 10 years after untreated syphilis, with personality changes, ataxia, blurred vision, headache, dizziness, and hearing loss. Pupillary response to light is lost (Argyll Robertson pupils), and there is loss of proprioception and vibration sense. Although this patient has a history of syphilis at age 20, he was treated and his present rapid plasma reagin test is negative. Therefore, neurosyphilis is unlikely.

How well did you know this?
1
Not at all
2
3
4
5
Perfectly
19
Q

A 22-year-old woman comes to the physician for her first physical health examination in several years. She says that she has always been in good health except for irregular menses in the past year. She does not take any medication or smoke. Her blood pressure is 137/80 mm Hg. Her height is 160 cm (63 in), and her weight is 83 kg (185 lb). Physical examination reveals a slight increase in upper lip and chin hair. She reports that she has been gaining weight since the age of 15 despite all attempts to both limit calorie intake and exercise. Which of the following is the most appropriate next step in management?

(A) No intervention needed at this time
(B) Explanation of risks of obesity and benefits of weight loss
(C) Laboratory investigations to exclude secondary causes of obesity
(D) Referral to weight reduction program for very-low-calorie diet
(E) Pharmacologic treatment with the serotonin reuptake inhibitor sibutramine

A

Respuesta: C

The correct answer is C. Obesity can be defined by using the nomograms based on the statistical studies of the National Center for Health Statistics on large population samples in the U.S. These data allow estimating the body mass index (BMI) from height (in inches) and weight (in pounds) [To calculate this index more rigorously, BMI = weight/(height)2 , where weight is in kilograms and height is in meters]. Most authors agree that obesity is present when the BMI is higher than 30 kg/m2. This patient has a BMI of 33 kg/m2. Once obesity is identified, the most appropriate next step is to determine whether obesity is secondary to underlying pathologic conditions. Physical examination and history should focus on detecting signs and symptoms of the three most common causes of secondary obesity, namely hypothyroidism, Cushing syndrome, and genetic conditions. In this case, menstrual irregularities and slight hirsutism are features that suggest the need to undertake additional investigations to rule out endocrinologic or genetic causes. Stein-Leventhal syndrome (in addition to those mentioned) should be considered in this case.

No intervention at this time (choice A) would not be appropriate. Active diagnostic and therapeutic interventions are needed for all patients whose BMI is >30 kg/m2 , and patients with BMI >27 kg/m2 should be encouraged to lose weight. In fact, it is well established that obesity is associated with increased risk of developing hypertension, diabetes, cardiovascular disease, cholelithiasis, pulmonary dysfunction, osteoarthritis, and some forms of cancer.

The physician should explain the risks of obesity and benefits of weight loss (choice B) to all overweight patients, although most of them are already painfully aware of the disadvantages inherent to obesity, with regards to both health risks and social stigma.

Treatment options for obese patients include decreasing calorie intake and/or increasing energy expenditure, variably combined with techniques of behavioral modification. A weight reduction program for a very-low-calorie diet (choice D) is an aggressive approach that replaces the whole daily food intake with a diet or a beverage containing no more than 800 kcal/day, 1 g protein/kg of body weight, plus all of the necessary vitamins and minerals. This diet is used as part of a comprehensive weight-loss program, in which the patient is closely monitored by a medical specialist to prevent possible serious adverse effects.

Pharmacologic treatment with the serotonin reuptake inhibitor sibutramine (choice E) has been approved by the FDA for treatment of obesity. However, the long-term effects of this therapy are still under investigation. Hypertension has been observed in some patients. In any case, this type of treatment should be used only after secondary obesity has been excluded.

How well did you know this?
1
Not at all
2
3
4
5
Perfectly
20
Q

A 55-year-old man presents to a physician because of a 2-month history of difficulty swallowing. At first, the difficulty was only with large bites of solid food, and he was able to limit it by taking smaller bites and washing them down with drinks. However, he now has trouble with small bites and liquids. He has a long history of heavy use of both alcohol and cigarettes. Esophagoscopy demonstrates a large, irregular polypoid mass that is nearly occluding the upper third of the esophagus. A biopsy of the tumor is most likely to show which of the following?

(A) Anaplastic squamous epithelial cells with numerous mitotic figures
(B) Large, lymphocytic cells with large, prominent nucleoli
(C) Mucin-producing glandular tissue with signet ring cells
(D) Small, lymphocytic cells with irregular nuclei and condensed chromatin
(E) Small, polygonal cells with neurosecretory granules

A

Respuesta: A

The correct answer is A. Cancers involving the upper third of the esophagus are usually squamous cell in origin; histologically, they are described as anaplastic squamous cells with numerous mitotic figures. Lower esophageal cancers may be either squamous cell carcinomas or adenocarcinomas (usually arising in Barrett esophagus). Esophageal cancer has a very poor prognosis (most patients die within 2 years) because the cancer usually has advanced through the esophageal wall by the time the patient presents with dysphagia. Because the esophageal wall is thin, it is easy for the cancer to penetrate to the level of the lymphatics (less than 1 mm), where metastasis can occur, or penetrate completely through the esophagus (2 to 4 mm) to directly involve nonresectable mediastinal structures, such as the aorta, heart, or vicinity of the carina of the bronchial tree. Predisposing factors for squamous cell carcinoma of the esophagus include alcohol and tobacco use, human papillomavirus, esophageal scarring (lye ingestion, irradiation), sclerotherapy, and chronic achalasia.

Large, lymphocytic cells with large, prominent nucleoli (choice B) describe large cell lymphoma, whereas small, lymphocytic cells with irregular nuclei and condensed chromatin (choice D) describe small cell lymphoma. Both of these cancers are unlikely to be found in this patient.

Mucin-producing glandular tissue with signet ring cells (choice C) is the classic description of adenocarcinoma, which is unlikely to be found in the upper third of the esophagus. It usually arises in patients who have had long-standing metaplastic changes of Barrett esophagus.

Small, polygonal cells with neurosecretory granules (choice E) describe small cell carcinoma of the lung, which is also called oat cell carcinoma. This cancer is strongly associated with cigarette smoking and usually presents as a central or hiker lung tumor.

How well did you know this?
1
Not at all
2
3
4
5
Perfectly
21
Q

A 19-year-old African American woman with sickle cell anemia has had palpitations and dizziness for 3 days. She works part-time in a daycare center. Her temperature is 38.1 C (100.5 F), pulse is 110/min, and respirations are 18/ min. The cardiac examination is significant for a systolic murmur heard best at the apex. Her lungs are clear, and her abdominal examination is unremarkable. Laboratory studies reveal a hemoglobin of 6.0 g/dL. A blood smear stained with Wright’s stain demonstrates Howell-Jolly bodies and the absence of reticulocytes. Which of the following is the most likely pathogen?

(A) Influenza virus
(B) Parainfluenza virus
(C) Parvovirus
(D) Salmonella
(E) Streptococcus pneumoniae

A

Respuesta: C

The correct answer is C. This patient is experiencing an aplastic crisis due to parvovirus infection. Parvovirus exposure is common in daycare centers. Individuals with sickle cell disease, like those with other chronic hemolytic diseases, are susceptible to infection with parvovirus. Patients usually present with weakness, lethargy, and severe anemia often preceded by a few days of nonspecific symptoms. These patients have intense reticulocytopenia, and the bone marrow contains no erythroid precursor cells, despite a normal myeloid series. A transient aplastic crisis due to parvovirus infection may produce life-threatening anemia and may require urgent transfusion. Note that Howell-Jolly bodies are consistent with asplenism.

Influenza virus (choice A) and parainfluenza virus (choice B) present with fever and systemic symptoms, including myalgia, headache, and malaise. Sudden development of severe anemia with reticulocytopenia is not seen in these infections.

Salmonella infection (choice D) is more common in sickle cell patients because of splenic hypofunction, but it presents either as typhoidal illness or diarrhea. Selective red cell aplasia is not a feature of Salmonella infection. Note that osteomyelitis due to

Salmonella is more common in sickle cell patients.
Infection due to Streptococcus pneumoniae (choice E), although common in sickle cell disease patients because of splenic hypofunction, more frequently manifests as pneumonia or, less commonly, as meningitis.

22
Q

A healthy 20-year-old man presents with a history of recurrent episodes of severe throbbing headache. The headaches are triggered or aggravated by noise and stress and last for several hours. The pain is frequently preceded by visual disturbances, such as luminous stars or zigzags of light, and accompanied by nausea and vomiting. His father experienced a similar form of headache. The patient has found no relief with aspirin and ibuprofen. Which of the following is the most appropriate drug treatment during the acute attacks?

(A) Acetaminophen
(B) Calcium-channel antagonists
(C) Carbamazepine
(D) Ergotamine
(E) Prednisone

A

Respuesta: D

The correct answer is D. The clinical picture is virtually
pathognomonic of migraine, especially considering the premonitory visual symptoms (the aura), the throbbing quality of the pain, and the positive family history. However, migraine does not always present classically. A common form of migraine may be more frequent, with diffuse (not unilateral) throbbing pain of moderate intensity, which is not preceded by visual disturbances or associated with nausea. Acute treatment is based on the administration of antimigraine drugs at the onset of symptoms. Effective drugs include ergotamine tartrate or similar compounds, and sumatriptan.

Prophylactic treatment is aimed at decreasing the frequency of
attacks. Calcium-channel blockers (choice B), for example, have been used for prophylaxis.

Acetaminophen (choice A) is very unlikely to provide relief in classic migraine, especially if aspirin and ibuprofen have already failed to do so.

Carbamazepine (choice C) has been found particularly helpful in the treatment of trigeminal neuralgia (tic douloureux), but not in migraine.

Prednisone (choice E) should be used to treat patients manifesting signs and symptoms strongly suggestive of giant cell arteritis (temporal arteritis), which may lead to blindness in the absence of effective anti-inflammatory therapy. However, temporal arteritis is much more common in the elderly and would not be in the differential diagnosis in this 20-year-old.

23
Q

A 55-year-old man consults a physician because of weight loss and severe abdominal pain of several months’ duration. The pain radiates to the mid-back and is slightly relieved when he assumes a bent forward position. On physical examination, the man appears emaciated, with mild jaundice. The liver edge is palpable and smooth; the liver depth is not increased. The clinician suspects pancreatic cancer. Which of the following tests is the most appropriate next step in diagnosis?

(A) CT scan
(B) MRI scan
(C) Ultrasound
(D) Arteriography
(E) Endoscopic retrograde pancreatography

A

Respuesta: A

The correct answer is A. Ductal carcinoma of the pancreas is often devastating because, in roughly 90% of cases, it presents late in the clinical course, when it is no longer resectable. Therefore, the first step when this disease is suspected on clinical grounds is a CT scan, which is now recommended as the most cost-effective method of both diagnosing and staging the tumor (i.e., determining whether it is potentially resectable). If unresectable disease is detected, then definitive tissue diagnosis (for choice of chemotherapy) can be made on the basis of CT-guided percutaneous needle aspiration or biopsy. If a potentially resectable lesion is identified, endoscopic ultrasound (not yet widely available) can be used to search for small metastatic lesions not visible with CT.

MRI (choice B) is no more accurate than CT in detecting and staging pancreatic cancer, and is more expensive.

Ultrasound (choice C) is still frequently used, but usually is followed by CT scan. Therefore, the current thinking is to eliminate this test and go directly to CT, since the ultrasound is less sensitive.

Arteriography (choice D) is rarely used and is primarily for
determining vascular invasion and tumor resectability.

Endoscopic retrograde pancreatography (choice E) is still commonly used in working up pancreatic cancer. However, this is an invasive procedure and would not be the first step in diagnosis.

24
Q

A 70-year-old man presents to his physician with complaints of blurred vision in his right eye along with intermittent loss of vision, which has been occurring for the past 3 days. He describes it as a “curtain passing vertically” across the visual field. He does not have any pain, fever, headache, nausea, or vomiting. He has a history of hypertension and diabetes and is a smoker. Current medications include captopril and twice-daily insulin. On examination, the conjunctivae are pink, the sclerae are clear, and the pupils bilaterally are 3-4 mm and reactive. Cranial nerves II-XII are intact, and there are no focal neurologic deficits. Which of the following is most likely diagnosis?

(A) Amaurosis fugax
(B) Diabetic retinopathy
(C) Retinal artery occlusion
(D) Retinal detachment
(E) Retinal vein occlusion

A

Respuesta: A

The correct answer is A. A sensation of a curtain passing across the visual field can be characteristic of either amaurosis fugax or retinal detachment (choice D). In this case, the phenomenon is transient, so amaurosis fugax is more likely than retinal detachment. Fleeting blindness is characteristically caused by retinal emboli from ipsilateral carotid disease. The visual loss is described as a curtain passing vertically across the visual field, with complete monocular visual loss lasting a few minutes. Amaurosis fugax may be due to cholesterol plaque release and is a precursor of retinal artery occlusion.

Diabetic retinopathy (choice B) is the most common cause of blindness in the U.S. in adults aged 20-65 years, but the clinical description favors a diagnosis of amaurosis fugax.

Retinal artery occlusion (choice C) is characterized by sudden profound visual loss. Pupils are unreactive to direct light, and there is a cherry-red spot on the fovea.

Retinal detachment (choice D) causes progressive, unilateral, and painless blurred/loss of vision in one eye. There is often a sensation of a curtain passing down over the eyes.

Retinal vein occlusion (choice E) produces a sudden decrease or loss of vision; the pupils react sluggishly to light. Younger patients may present with near normal vision, whereas older patients may have significant obscuration.

25
Q

A study by McGinnis and Foege, published in the Journal of the American Medical Association in 1993, showed that up to 43% of deaths occurring annually in the U.S. are potentially preventable. Which of the following is an example of a program of primary (versus secondary or tertiary) prevention?

(A) Annual mammography for women older than 45
(B) Controlling blood lipid levels
(C) Controlling hypertension
(D) Increasing cigarette taxes
(E) Pap smear screening
(F) Prophylactic aspirin after myocardial infarction

A

Respuesta: D

The correct answer is D. Preventive programs are aimed at reducing the likelihood that a disease develops or worsens. Preventive programs can be divided into three types. Primary prevention attempts to prevent disease before it develops. Examples of primary prevention include all those programs aimed at reducing the risk of specific diseases: avoidance of cigarette smoking to prevent lung cancer and cardiovascular disease, immunizations to reduce the risk of infectious illnesses, and so on. Often, public health programs for primary prevention are more effective than an individual-based approach. For example, increasing taxes on cigarettes has been shown to reduce cigarette smoking more effectively than has advice to quit smoking given by physicians. Nevertheless, the beneficial role of physicians in individual-based prevention is undeniable.

Secondary prevention is based on early detection of an existing pathologic state before it causes damage to the organism. Examples of secondary prevention include the following: annual mammography for women older than 45 (choice A) to detect breast cancer in early stages, controlling hypertension (choice C) and blood lipid levels (choice B) to reduce the risk of atherosclerotic complications, and Pap smear screening (choice E) to detect early stages of cervical cancer or its histologic precursors.

Tertiary prevention is based on rehabilitative interventions or other methods to improve the clinical evolution of a disease that has already developed. Prophylactic aspirin after myocardial infarction (choice F) is a classic example of tertiary prevention.

26
Q

A 58-year-old man with a 12-year history of type 2 diabetes mellitus comes to the physician because of an ulcer in his right foot. Physical examination reveals a 1-cm irregular ulceration over the right metatarsal head, surrounded by an area of black gangrenous skin. The patient is admitted to the hospital and undergoes amputation of the right forefoot. Which of the following measures would have been most effective in preventing this complication?

(A) Appropriate instructions on self-care of the feet
(B) Doppler examination of the lower extremities
(C) Neurophysiologic and electromyographic studies
(D) Local application of platelet-derived growth factor
(E) Prophylactic treatment with cholesterol-lowering agents

A

Respuesta: A

The correct answer is A. Diabetic patients are particularly prone to gangrene of the feet. Sensory loss due to peripheral neuropathy, small vessel disease leading to ischemia, and secondary infections are the pathogenetic factors underlying this pathology. Diabetes is the leading cause of nontraumatic amputations. In addition to strict glycemic control to prevent vascular and neurologic complications, the most effective method of prevention is self-care. The patient should receive instructions on daily foot self-examination (to look for abrasions and blisters), wearing appropriate shoes, cutting toenails straight across, and avoiding barefoot walking.

Doppler examination of the lower extremities (choice B) assesses vascular insufficiency. However, since the arteries may become rigid because of calcific atherosclerosis, falsely elevated blood pressure readings may be obtained with this test.

Periodic neurophysiologic and electromyographic studies (choice C) are not useful to prevent foot complications. Neurophysiologic and electromyographic studies may be performed when there are symptoms of peripheral neuropathy.

Local application of platelet-derived growth factor (choice D) has some efficacy in treating nonhealing ulcers refractory to debridement and antibiotic therapy.

Prophylactic treatment with cholesterol-lowering agents (choice E) should be considered for diabetic patients with high cholesterol levels to prevent or slow atherosclerotic change.

27
Q

A 35-year-old woman has had erythematous, round, scaling papules for several years. The lesions are 5-10 mm in diameter and show follicular plugging. Although the lesions are generalized in distribution, the highest number are on the malar prominences, bridge of the nose, scalp, and external auditory canals. Biopsy of the lesions is consistent with either discoid lupus erythematosus or systemic lupus erythematosus. Which of the following additional findings would most strongly tend to exclude discoid lupus?

(A) Alopecia of the scalp
(B) Anti-ds DNA
(C) Noncontracting scars
(D) Positive antinuclear antibody test
(E) Sun sensitivity

A

Respuesta: B

The correct answer is B. The skin manifestations of discoid lupus and systemic lupus erythematosus can be indistinguishable. Although the two conditions probably represent extremes of a spectrum, it is convenient to separate them into discoid lupus, which has none to minimal systemic effects, and systemic lupus, which often has prominent effects on other organ systems. A real clinical question arises in newly diagnosed patients, who may have either just skin manifestations or skin manifestations that are the earliest feature of what will become systemic disease. The identification of antibodies to double-stranded DNA, which are more specific than antinuclear antibodies, can exclude discoid lupus, since they are not encountered in this variant. Unfortunately, not all patients with systemic lupus develop these antibodies, so a negative result is not helpful. In these cases, the progress through time of the disease will permit its classification.

Alopecia, or hair loss, of the scalp (choice A) can be seen in both conditions.

Noncontracting scars (choice C), usually at the centers of old lesions, can be seen in both conditions.

A positive antinuclear antibody test (choice D) is almost always seen in systemic lupus, but can also be seen in up to 10% of discoid patients.

Sun sensitivity (choice E) is seen in both conditions.

28
Q

A 25-year-old man with acute myelogenous leukemia is undergoing chemotherapy. One week after his therapy, he presents with a headache, fever, and confusion. On physical examination, he has nuchal rigidity, Kernig sign, and photophobia. Laboratory results are notable for a white count of 1000 mm3, hematocrit of 25%, and a differential of 10% neutrophils and 5% lymphocytes. Lumbar puncture is consistent with meningitis. Which of the following is the most likely pathogen?

(A) Bacteroides fragilis
(B) Haemophilus influenzae
(C) Pseudomonas aeruginosa
(D) Staphylococcus aureus
(E) Toxoplasma gondii

A

Respuesta: C

The correct answer is C. The patient is neutropenic with an absolute neutrophil count (ANC) of 100 (1000 × 10%). He has symptoms of meningitis. By definition, a patient with an ANC less than 500 is neutropenic. Such patients are susceptible to gram-negative bacteria, such as Pseudomonas, and would be treated with IV ceftazidime.

Bacteroides (choice A) is an anaerobic agent and, along with Clostridium, is a typical pathogen in abscesses and gangrene. Infection with Bacteroides would require treatment with IV metronidazole.

Haemophilus (choice B) is the most common cause of meningitis in children. It is a gram-negative organism and would require treatment with an IV antibiotic, such as a third-generation cephalosporin.

Staphylococcus (choice D) could cause meningitis after a penetrating wound and would require treatment by a IV penicillinase-resistant antibiotic, such as nafcillin or vancomycin.

Toxoplasma (choice E) is a serious CNS infection in patients with HIV. Patients with Toxoplasma encephalitis will have focal or multifocal abnormalities demonstrable on CT or MRI. Common neurologic symptoms include seizures, meningoencephalitis, and headaches.

29
Q

A healthy 25-year-old woman comes to the physician for a health maintenance examination. Her blood pressure is 126/82 mm Hg, pulse is 75/min and regular, and respirations are 14/min. She denies any significant health problems and exercises regularly. Cardiac auscultation reveals a low-pitch grade III/VI mid-diastolic murmur near the apex. It begins with a snap and ends before the first heart sound. The lungs are clear to auscultation. An ECG shows no abnormalities. Which of the following is the most appropriate next step in diagnosis?

(A) Antistreptolysin O titers
(B) Doppler ultrasound
(C) Echocardiography
(D) Radionuclide angiography
(E) Cardiac catheterization

A

Respuesta: C

The correct answer is C. The clinical presentation is consistent with mitral stenosis, with its typical opening snap followed by a diastolic “rumbling” murmur. Echocardiography is the technique of choice to evaluate mitral valve abnormalities; it can confirm an auscultatory diagnosis of mitral stenosis. Rheumatic fever continues to be one of the most frequent causes of mitral valve stenosis. Rheumatic fever is usually a sequela of a pharyngitis due to group A beta-hemolytic streptococci.

Elevated antistreptolysin O titers (choice A) are used to confirm recent streptococcal infection but would have no value in the assessment of long-term complications of rheumatic heart disease.

Doppler ultrasound (choice B) can give quantitative estimates of transvalvular gradients and mitral valve area, but this is usually performed after echocardiography.

Radionuclide angiography (choice D) is mainly used to assess left and right ventricular ejection fraction. This technique also allows the study of segmental wall motion, and can be used to estimate valvular regurgitation and measure pulmonary-to-systemic flow ratio in left-to-right shunts. It is not commonly used in the diagnosis of mitral valve disease. Most of the information needed in cases of mitral stenosis can be obtained by clinical and usually echocardiographic studies. Thus, cardiac catheterization (choice E) is not necessary, unless surgery is being considered and additional information is necessary.

30
Q

A 30-year-old man presents to the emergency department with severe headache, visual changes, and palpitations. His temperature is 37.2 C (100 F), blood pressure is 190/130 mm Hg, pulse is 130/min, and respirations are 17/min. The remainder of the physical examination is unremarkable except for clamminess of the hands and increased sweating on the face. The patient’s hypertension is treated. A 24-hour urine specimen demonstrates elevated metanephrine, vanillylmandelic acid (VMA), and homovanillic acid (HVA). On questioning, the patient notes that endocrine problems have been very common in his family. This patient is most likely at significantly increased risk of developing which of the following thyroid diseases?

(A) Graves disease
(B) Hashimoto disease
(C) Medullary carcinoma
(D) Multinodular goiter
(E) Papillary carcinoma

A

Respueta: C

The correct answer is C. The patient has a pheochromocytoma, which has produced the severe hypertension with excess urinary metabolic products of epinephrine and norepinephrine demonstrated in the 24-hour urine collection. The family history of “endocrine problems” suggests the possibility of multiple endocrine neoplasia (MEN), of which medullary carcinoma of the thyroid may be a part, particularly in type IIa (Sipple syndrome) and type IIB (mucosal neuroma syndrome).

Graves disease (choice A) and Hashimoto disease (choice B) can be associated with other autoimmune diseases, but are not associated with the MEN syndromes.

Multinodular goiter (choice D) can be associated with iodine deficiency.

Papillary carcinoma of the thyroid (choice E) can be associated with history of neck irradiation.

31
Q

A 12-year-old boy presents with acute onset of morbilliform rash, fever, malaise, and oliguria. These manifestations began 1 week after starting treatment with ampicillin for streptococcal pharyngitis. His temperature is 38.8 C (102 F), blood pressure is 115/76 mm Hg, pulse is 95/min, and respirations are 16/min. Urinalysis shows microhematuria, leukocyturia with numerous eosinophils, and occasional white blood cell casts. Proteinuria is absent. Blood studies show elevated antistreptolysin titers and moderate eosinophilia. BUN is 42 mg/dL, and serum creatinine is 2.5 mg/dL. Which of the following is the most likely diagnosis?

(A) Acute interstitial nephritis
(B) Acute pyelonephritis
(C) Acute tubular necrosis
(D) Henoch-Schönlein purpura
(E) Post-streptococcal glomerulonephritis

A

Respuesta: A

The correct answer is A. Acute interstitial nephritis is probably the second most common cause of intrinsic renal failure (after acute tubular necrosis). The most frequent causative factors are drugs, including penicillins (especially methicillin), cephalosporins, sulfonamides, NSAIDs, rifampin, and phenytoin. Coexistence of skin rash, eosinophils in the urine, and blood eosinophilia is an important diagnostic clue.

Acute pyelonephritis (choice B) often develops in the setting of some underlying urologic diseases. In children, the most common underlying factor is vesicoureteral reflux. High fever, flank pain, and abundant pyuria are the main clinical manifestations.

Acute tubular necrosis (choice C) develops as a result of severe hypoxia, prolonged prerenal azotemia, exogenous nephrotoxic agents (e.g., aminoglycosides), or endogenous substances (e.g., myoglobinuria, hemoglobinuria, or severe hyperuricemia). Granular casts, but not eosinophils, are found in the urine.

Henoch-Schönlein purpura (choice D) is due to a small-vessel vasculitis secondary to IgA deposition. It manifests with some combination of purpura (due to dermal vessel involvement), nephritic syndrome, abdominal pain, melena, and arthralgia.

Poststreptococcal glomerulonephritis (choice E) is associated with a classic nephritic syndrome, which manifests with pedal and periorbital edema, hypertension, hematuria, and mild proteinuria. Red blood cell casts are seen in the urine.

32
Q

An 18-year-old man presents with pain in his left knee and right ankle. There is no history of trauma. The young man states that he has not felt well since recovering from a 2-week diarrheal illness 1 month ago. On physical examination, his temperature is 38.1 C (100.6 F), blood pressure is 100/70 mm Hg, pulse is 76/min, and respirations are 16/min. He has bilateral conjunctivitis. His right ankle and left knee are swollen, erythematous, warm, and tender. There is a small effusion present in the knee. His right Achilles tendon is also erythematous and tender. Synovial fluid from his knee shows a white blood cell count of 10,000/mm3 with 80% neutrophils. No organisms are seen on Gram stain. Which of the following is the most likely diagnosis?

(A) Crohn disease
(B) Felty syndrome
(C) Gout
(D) Juvenile rheumatoid arthritis
(E) Reiter syndrome
(F) Septic arthritis

A

**Respuesta: E

The correct answer is E. This young man has developed asymmetric, noninfectious polyarthritis, conjunctivitis, and Achilles tendinitis following a self-limited diarrheal episode. This is called Reiter syndrome, or reactive arthritis. The other features of this syndrome are urethritis, circinate balanitis, keratoderma blennorrhagica, anterior uveitis, and an association with HLA-B27.

Abdominal pain and chronic diarrhea are the main features of Crohn disease (choice A). Intestinal malabsorption and significant weight loss may be present. Arthritis is seen in some patients with Crohn disease and affects lumbosacral and sacroiliac joints, as well as peripheral joints. However, Crohn disease is usually not characterized by an acute, self-limited diarrheal episode.

Felty syndrome (choice B) consists of chronic rheumatoid arthritis, splenomegaly, neutropenia, and, on occasion, anemia and thrombocytopenia. It is most common in individuals with long- standing disease. These patients frequently have high titers of rheumatoid factor, subcutaneous nodules, and other manifestations of systemic rheumatoid disease.

In gout (choice C), the initial attack typically affects only one joint, most commonly the first metatarsal joint. Monosodium urate crystals in the joint fluid, which appear long, needle shaped, and negatively birefringent, are diagnostic. The synovial fluid leukocyte count tends to be higher, in the 25,000 to 50,000/mm3 range.

Juvenile rheumatoid arthritis (choice D) is a particular arthritis involving mainly the knees and hips. Achilles tendinitis is not a feature of this disease.

Septic arthritis (choice F) should always be considered with sudden- onset inflammatory arthritis in one or more joints. It can lead to rapid destruction if it is untreated. Septic arthritis is classified as gonococcal or nongonococcal, and it is presumptively diagnosed when synovial fluid has a white blood cell count over 50,000/mm3 and over 90% neutrophils. Organisms are often found on Gram stain of synovial fluid.

33
Q

A 25-year-old schizophrenic patient presents with painful oral ulcers. He was started on chlorpromazine treatment 3 months ago. He is currently afebrile, and there is no evidence of significant somatic disease on physical examination. Blood analyses show:

Hematocrit: 45%
Platelet count: 320,000/mm3
Leukocyte count: 400/mm3

Differential:
*Lymphocytes: 85%
*Monocytes 10%
*Neutrophils 5%

Morphology of red and white blood cells is normal on peripheral blood smears. Which of the following is the most appropriate next step in management?

(A) Discontinue current pharmacologic treatment
(B) Treat stomatitis by topical corticosteroids
(C) Treat with myeloid growth factors (G-CSF and GM-CSF)
(D) Admit patient for broad spectrum antibiotic treatment
(E) Perform bone marrow aspirate examination

A

Respuesta: A

The correct answer is A. This patient presents with clinical manifestations consistent with drug-induced neutropenia. Neutropenia is defined as a blood cell count lower than 1500/mm3 . Neutropenic patients are susceptible to bacterial and fungal infections. Drugs that may cause neutropenia include chlorpromazine, sulfonamides, procainamide, methimazole, propylthiouracil, penicillins, cephalosporins, and multiple chemotherapeutic agents. Clinical severity may vary considerably in relation to the degree of neutropenia. Sore throat and oral ulcers (stomatitis) are the mildest signs, but life-threatening infections may be the presenting manifestations. If signs of infection are absent, the patient may be followed on an outpatient basis, and, of course, suspected drugs should be immediately discontinued.

Topical corticosteroid therapy for stomatitis (choice B) is not useful in this case. It is occasionally beneficial in aphthous ulcers, i.e., small but painful erosions of the buccal mucosa. Oral ulcers associated with neutropenia will resolve as soon as leukocyte counts return to normal levels.

Treatment with myeloid growth factors (G-CSF and GM-CSF; choice C) may be used to hasten recovery of bone marrow in patients with neutropenia secondary to chemotherapy.

Admitting the patient for broad-spectrum antibiotic treatment (choice D) is not recommended unless there is fever or other signs of infection. Hospitalization may indeed be more risky than useful to neutropenic patients, since it exposes them to nosocomial infections.

Examination of bone marrow aspirate (choice E) is not useful in this case, since both the condition and its underlying cause (i.e., neutropenia due to chlorpromazine toxicity) are relatively obvious, and are reversible with discontinuation of the drug.

34
Q

A 60-year-old man presents with shortness of breath and dull left- sided chest pain. Examination reveals decreased breath sounds on the left. A chest x-ray film is consistent with a large left-sided pleural effusion, for which the patient undergoes thoracentesis. The fluid is slightly turbid with a pH of 7.2, a white count of 60,000/mm3 , an RBC count of 15,000/mm3 , an LDH of 400 IU/L, and a serum LDH of 500 IU/L (normal 50-150 IU/L). A repeat chest x-ray film reveals a pneumonia in the right middle lobe. A pleural fluid Gram stain shows multiple gram-positive diplococci. Which of the following is the most appropriate next step in management?

(A) Ampicillin
(B) Diuresis
(C) Pleural biopsy
(D) Pleurodesis
(E) Chest tube insertion

A

Respuesta: E

The correct answer is E. Pleural effusions may be transudative or exudative. This effusion is exudative since the pleural fluid/serum LDH ratio is greater than 0.6. Causes of exudates include parapneumonic effusion, tuberculosis, pulmonary infarct, malignancy, and bacterial infection. Organisms in the fluid, a pleural fluid leukocyte count of 50,000, and a low pH constitute a complicated parapneumonic effusion. Such collections tend to loculate and form adhesions if not immediately drained with a chest tube.

This infection may prove too tenacious to eradicate with antibiotic coverage alone (choice A). The risk of mortality increases with age and extent of hemodynamic compromise. This patient probably has a streptococcal infection and would need coverage with a penicillin in association with chest tube placement.

Diuresis (choice B) would be an option if this patient had heart failure that was compromising his respiratory status, but such effusions are simple transudates.

Biopsy (choice C) will not be helpful in the diagnosis in this patient with an obvious parapneumonic effusion; thoracentesis must be performed, and the fluid examined. A common complication of a biopsy is pneumothorax.

Pleurodesis (choice D) might be an option if this were a malignancy and the patient had recurring effusions, compromising his pulmonary function. This is often achieved by injecting talc or bleomycin in the pleural space, thereby causing adhesions between the two layers of pleura and preventing future fluid collections.

35
Q

A 46-year-old housewife from Connecticut presents with complaints of malaise, arthritis, and a rash on her left thigh. ELISA and Western blot confirm infection with Borrelia burgdorferi. A 30- day course of oral doxycycline with follow-up in 1 week is prescribed. The patient cancels the follow-up appointment and returns 3 months later complaining of facial droop, confusion, daily fevers, and tingling in her hands and feet. She states that she felt markedly better after 2 weeks on the antibiotics and therefore discontinued her medication. Which of the following is the most appropriate next step in management?

(A) A 14-day course of chloramphenicol
(B) A 14-day course of IV ceftriaxone
(C) A 28-day course of IV vancomycin
(D) Prescribe a repeat 3-day course of oral doxycycline
(E) Prescribe a 30-day course of oral amoxicillin

A

Respuesta: B

The correct answer is B. This patient now has neurologic manifestations of Lyme disease. Approximately 15% of patients at some point develop frank neurologic abnormalities, including meningitis, encephalitis, chorea, cranial neuritis (including bilateral facial palsy), motor and sensory radiculoneuritis, or mononeuritis multiplex. The usual pattern is fluctuating meningoencephalitis with superimposed cranial nerve (particularly facial) palsy and peripheral radiculoneuropathy, but Bell palsy may occur alone. The best treatment is admission to the hospital for IV ceftriaxone (2 g daily for 10-21 days). An alternative is IV penicillin G (20 million units a day for 10-20 days).

Chloramphenicol (choice A) and vancomycin (choice C) are not appropriate drugs for the treatment of Lyme disease. Oral antibiotic therapy (choices D and E) is not appropriate for treating advanced neurologic Lyme disease.

36
Q

A 65-year-old man comes to the physician because of an increasingly severe tremor that affects the right hand. The tremor is particularly marked at rest and disappears when the limb is in movement. The man’s speech is soft but not monotonous. There is increased resistance when the arms or neck are passively flexed. Sensation and muscle strength appear intact. Short-term memory is preserved. The patient’s blood pressure is 134/82 mm Hg, temperature is 37 C (98.6 F), pulse is 70/min, and respirations are 10/min. The patient has a history of a previous episode of narrow- angle glaucoma. Which of the following drugs should be avoided in the treatment of his neurologic condition?

(A) Amantadine
(B) Benztropine
(C) Bromocriptine
(D) Levodopa
(E) Selegiline

A

Respuesta: B

The correct answer is B. The clinical picture is consistent with Parkinson disease at a relatively early stage. Resting tremor may be unilateral at first. Anticholinergic drugs, such as benztropine, are frequently used initially and are effective in alleviating tremor and rigidity. The key to the correct answer is the fact that the patient’s history includes narrow-angle glaucoma. Use of anticholinergic drugs may lead to an acute increase of intraocular pressure in predisposed individuals and precipitation of narrow-angle glaucoma. Other contraindications to the use of anticholinergics include prostatic hyperplasia (or symptoms of urinary retention) and gastrointestinal obstruction (or severe constipation).

Amantadine (choice A) is also used for mild parkinsonism, although its mechanism of action is unclear. Depression, postural hypotension, and cardiac arrhythmias are the most serious adverse effects.

Bromocriptine (choice C) is one of the dopamine agonists used for Parkinson disease. This drug was used before the introduction of levodopa; currently, it is sometimes used in association with low doses of levodopa-carbidopa. Bromocriptine is contraindicated in patients with neuropsychiatric disturbances, recent myocardial infarction, or peptic ulcer.

Levodopa (choice D) is the drug of choice for treatment of Parkinson disease. Nausea, vomiting, and hypotension are the most common side effects.

Selegiline (choice E) is an inhibitor of monoamine oxidase B. It is used as an adjunctive treatment along with levodopa. Available evidence suggests that selegiline might be effective in retarding progression of the disease.

37
Q

A patient presents to a physician with severe jaundice. Physical examination reveals a nodular, enlarged liver. In addition to the generalized nodularity of the liver, the physician can feel one nodule that is much larger than the others. CT of the abdomen confirms multinodular cirrhosis and demonstrates a 7-cm mass near the lower border of the liver. CT-guided biopsy of this mass shows a malignant tumor derived from hepatic parenchymal cells. Which of the following risk factors is most strongly associated with the development of this tumor?

(A) Aflatoxin exposure
(B) Hemochromatosis
(C) Hepatitis B virus infection
(D) Opisthorchis infection
(E) Thorotrast exposure

A

Respuesta: C

The correct answer is C. The tumor is a hepatocellular carcinoma, which usually develops in the setting of cirrhosis because of a variety of damaging agents. By far the most commonly implicated etiologic factor, both worldwide and in the U.S., is infection with hepatitis B or C. Other important risk factors include alcohol abuse, hemochromatosis, and aflatoxin exposure.

Aflatoxin (choice A) is a fungal toxin found in contaminated bean products, including soy beans and soy products (e.g., soy sauce).

Hemochromatosis (choice B) is a disease of disordered iron metabolism, which particularly damages liver, pancreas, heart, and skin.

Opisthorchis (choice D) is a liver fluke that infects the biliary tract and predisposes for cholangiocarcinoma, not hepatocellular carcinoma.

Thorotrast (choice E) is a radiologic contrast medium that is no longer used because it predisposed for cholangiocarcinoma, not hepatocellular carcinoma.

38
Q

A 37-year-old florist comes to the employee health clinic for a routine evaluation. He is healthy and without complaints. Five units of tuberculin protein (PPD) is injected intradermally. He returns to the clinic 48-72 hours later. Which of the following would indicate a positive reaction in this patient?

(A) 5 mm of erythema and 5 mm induration
(B) 10 mm of erythema and 5 mm induration
(C) 15 mm of erythema and 5 mm induration
(D) 15 mm of erythema and 15 mm induration
(E) 20 mm of erythema and 10 mm induration

A

Respuesta: D

The correct answer is D. The tuberculin skin test is usually applied to the forearm. Reaction should be read measuring the transverse diameter of induration as detected by gentle palpation at 48-72 hours. Reaction of >15 mm is considered a positive test in patients from a low-risk population. The degree of erythema is unimportant.

An induration of 5 mm (choices A, B, and C) is considered positive in patients at high risk to be infected, i.e., immunocompromised patients, and in household contacts of tuberculosis patients.

An area of induration measuring 10 mm (choice E) is considered positive only in patients from population groups at elevated risk of tuberculosis, i.e., health care workers.

39
Q

A 18-year-old woman comes to medical attention because of fever and a red papule on her left forearm, which developed 1 week after being scratched by her cat. She has had temperatures to 38.5 C (101.3 F) and malaise for 2 days. Examination reveals enlarged and tender lymph nodes in the epitrochlear and axillary regions. Which of the following is the most likely pathogen?

(A) Bartonella henselae
(B) Bartonella quintana
(C) Calymmatobacterium granulomatis
(D) Chlamydia psittaci
(E) Pasteurella multocida

A

Respuesta: A

The correct answer is A. Cat-scratch disease is an acute infection due to Bartonella henselae that is transmitted to humans by a cat scratch or bite. A papule or an ulcer develops at the site of the scratch/bite, followed 1-2 weeks later by fever, malaise, and regional lymphadenopathy. The disease is self-limiting and does not require any treatment. Rarely, biopsies of lymph nodes are necessary to establish a diagnosis. Affected lymph nodes will exhibit a necrotizing granulomatous reaction with characteristic stellate-shape areas of necrosis. B. henselae causes bacillary angiomatosis in immunocompromised patients.

Bartonella quintana (choice B) is the etiologic agent of trench fever, which is transmitted by the human body louse.

Calymmatobacterium granulomatis (choice C) causes a sexually transmitted disease known as granuloma inguinale. A slowly enlarging, painless ulcer develops at the inoculation site, followed by granulomatous inflammation in the inguinal lymph nodes and, subsequently, scarring and adhesions.

Chlamydia psittaci (choice D) is the etiologic agent of psittacosis, transmitted by infected birds (parrots, parakeets, pigeons, and others). The disease manifests 1-2 weeks after exposure. It consists of atypical pneumonia indistinguishable from that caused by either viruses or bacteria.

Pasteurella multocida (choice E) is part of the normal mouth flora of cats and dogs. It is the most common pathogen causing early infection secondary to cat and dog bites. This manifests within 24 h after the bite with local swelling and pain, regional lymphadenopathy, and fever.

40
Q

A 56-year-old woman with a long history of painful osteoarthritis of the hip and lower back comes to medical attention because of polyuria for 3 months. She denies any previous urinary tract infection or renal disease. Her blood pressure is 135/80 mm Hg. Urine dipstick test shows hematuria and mild proteinuria. Blood studies reveal mild microcytic anemia, hyperkalemia, and normal uric acid levels. Ultrasonography shows kidneys of normal size. Intravenous pyelography (IVP) demonstrates the presence of characteristic “ring shadow” defects at the tips of renal papillae. Which of the following is the most likely cause of this condition?

(A) Analgesic nephropathy
(B) Lead exposure
(C) Multiple myeloma
(D) Obstructive uropathy
(E) Polycystic kidney disease
(F) Vesicoureteral reflux

A

Respuesta:

The correct answer is A. The long history of osteoarthritis should suggest chronic analgesic abuse as the underlying etiology of this renal condition, which is chronic tubulointerstitial nephritis. Analgesic nephropathy affects patients who consume significant amounts of aspirin, NSAIDs, phenacetin, or acetaminophen for at least 3 years. Clinical surveys have shown that patients often underestimate the amount of analgesics that they ingest daily. Note the main diagnostic clues of chronic tubulointerstitial nephritis: progressive polyuria because of inability of the renal tubules to concentrate urine, hyperkalemia, presence of radiologic signs of papillary necrosis (often associated with this condition), concomitant microhematuria, and mild proteinuria. Microcytic anemia may develop because of gastrointestinal blood loss secondary to the same drugs.

Lead exposure (choice B) is now a rare cause of chronic tubulointerstitial nephritis in the U.S. Lead intoxication also results in impaired tubular secretion of uric acid and hyperuricemia (note normal uric acid levels in this case); consequently, “saturnine” gout may ensue. Peripheral neuropathy may also manifest, leading to wrist drop. Suspect lead intoxication in alcoholics who drink “moonshine” alcohol prepared in old automobile radiators.

Multiple myeloma (choice C) is associated with bone involvement that manifests with pain and pathologic fractures. The resulting monoclonal gammopathy may cause renal damage. These patients are generally the elderly and may also present with a normochromic, normocytic anemia and hypercalcemia.

Obstructive uropathy (choice D) is another frequent cause of tubulointerstitial nephritis and papillary necrosis. It is commonly accompanied by an underlying disease, such as urolithiasis or prostatic hyperplasia. This patient does not have a history of renal disease.

Polycystic kidney disease (choice E) is an inherited renal disorder, presenting as multiple bilateral cysts that increase renal size and reduce functioning renal tissue. Patients may develop renal failure in the fourth to sixth decade of life. Abdominal pain, hypertension, hematuria, and impaired concentrating ability of the kidneys are common findings. Multiple bilateral cysts in the renal parenchyma and enlarged kidneys are seen on ultrasound and intravenous pyelography.

Vesicoureteral reflux (choice F) manifests in children and young adults with recurrent urinary tract infections. It is associated with hydronephrosis (dilatation of renal pelvis) and would not present for the first time in a 56-year-old.

41
Q

An 82-year-old woman is accompanied to the physician by her daughter because of repeated falls without apparent cause. The patient reports that she fell to the ground because of a sudden loss of strength in her legs without losing consciousness or feeling dizzy. She lay on the floor for a few minutes until she recovered strength and became able to stand up and walk again. She is otherwise in good health and takes alendronate for osteoporosis. Examination reveals mild resting tremor of her hands, but there is no rigidity or slowing of movements. Her blood pressure is 125/80 mm Hg, pulse is 68/min and regular, and respirations are 13/min. On auscultation, a bruit is heard over the right carotid artery. Which of the following is the most likely cause of this patient’s falls?

(A) Adverse drug reaction
(B) Lateral medullary infarction
(C) Parkinson disease
(D) Postural hypotension
(E) Transient ischemia in the carotid territory
(F) Transient vertebrobasilar ischemia

A

Respuesta: F

The correct answer is F. The clinical symptomatology is strongly suggestive of “drop attacks” resulting from transient ischemia in the vertebrobasilar territory. Ischemia of the pyramidal tract in the brainstem is the most probable pathogenetic mechanism. Transient ischemic attacks (TIAs), by definition, last less than 24 hours (usually less than 1 hour). TIA is often a harbinger of stroke, especially when involving the carotid circulation.

Adverse drug reaction (choice A) must be considered as a potential etiology of frequent falls in an elderly patient, especially when hypotension is the suspected mechanism and there is a history of antihypertensive medication. The most common adverse effect of alendronate is esophagitis.

Lateral medullary infarction (choice B) follows occlusion of the vertebral or posterior inferior cerebellar artery. Manifestations include ataxia, vertigo, nystagmus, impaired pain and temperature sensation on the ipsilateral face and contralateral body, dysphagia, and hoarseness.

Parkinson disease (choice C) leads to resting tremor, rigidity, and bradykinesia. Falls due to Parkinson disease result from loss of postural reflexes, which can be assessed by the pull-test.

Postural hypotension (choice D) is a frequent cause of falls in the elderly and is often the result of medication. Typically, the patient reports a light-headed sensation on standing or getting up from bed, and the fall may be accompanied by a transiently obtunded consciousness. The carotid bruit in this case is probably secondary to atherosclerotic stenosis of the carotid artery but not to the patient’s symptoms.

Transient ischemia in the carotid territory (choice E) manifests with motor deficits, sensory symptoms, or alterations in language expression or comprehension (aphasia).

42
Q

A 39-year-old man presents to the emergency department with acute onset of shortness of breath, hemoptysis, and left-sided pleuritic chest pain. His past medical history includes medication- controlled asthma, peptic ulcer disease, and a recent onset of idiopathic nephrotic syndrome. His blood pressure is 180/100 mm Hg, pulse is 110/min, and respirations are 28/min. Cardiac and lung examinations are normal. Laboratory data are remarkable for a serum lactate dehydrogenase of 300 U/L. An ECG shows sinus tachycardia, prominent S waves in lead 1, inversions of the T wave, and a prominent Q wave in lead III. Which of the following is the most likely cause of the chest pain?

(A) Aortic dissection
(B) Esophageal spasm
(C) Myocardial infarction
(D) Pneumonia
(E) Pulmonary embolism
(F) Variant angina

A

Respuesta: E

The correct answer is E. Clinical features suggestive of pulmonary embolism in this patient are pleuritic chest pain, hemoptysis, tachycardia, tachypnea, and elevated serum lactate dehydrogenase (suggestive of lung infarction). Individuals with nephrotic syndrome are at increased risk of pulmonary embolism because of an underlying hypercoagulable state. Possible mechanisms responsible for the underlying hypercoagulability include loss of anticoagulant proteins in the urine and intravascular volume depletion. The ECG findings are indicative of acute cor pulmonale, which may mimic inferior myocardial infarction (MI); however, an inferior wall MI is characterized by prominent Q waves and ST segment elevations in leads II, III, and AVF.

Aortic dissection (choice A) causes severe, tearing chest pain radiating to the back. Loss or decrease of a peripheral pulse, new- onset aortic insufficiency, and pericardial tamponade are possible physical findings of aortic dissection.

Esophageal spasm (choice B) can cause retrosternal chest pain and is usually associated with a history of dysphagia. The pain is typically substernal and usually not pleuritic in nature. It accounts for about 10% of noncardiac causes of chest pain.

Myocardial infarction (choice C) classically presents with retrosternal chest pain that may radiate to the left arm, neck, or jaw. The pain is described as a dull ache or heaviness in the chest and may be associated with dyspnea, diaphoresis, light-headedness, and nausea.

Pneumonia (choice D) may cause unilateral pleuritic chest pain if there is pleuritis or pleural effusion complicating the pneumonia. However, the patient would have fever and cough, indicating the presence of infection.

Variant angina (choice F) presents with chest pain or pressure at rest and the classic transient elevation of the S-T segment on ECG. Reduced coronary blood flow results from transient coronary spasm. Variant angina is not associated with elevated serum LDH or Q-and T-wave inversions on ECG.

43
Q

A 45-year-old woman with systemic lupus erythematosus (SLE) comes to the physician for a routine checkup. Her condition has been stable for several years, and she currently is not taking any medication. Blood chemistry studies and hematologic parameters are remarkable for a blood urea nitrogen (BUN) of 23 mg/dL, a creatinine of 1.6 mg/dL, and a mild normocytic anemia. The erythrocyte sedimentation rate is 18 mm/min. Urinalysis shows microhematuria and mild proteinuria. Which of the following is the most appropriate next step in management?

(A) Repeat urinalysis at next routine examination
(B) Sequential serum complement and ANA studies
(C) Treatment with corticosteroids
(D) Treatment with cyclophosphamide
(E) Renal biopsy

A

Respuesta: C
Vero: E

44
Q

A 62-year-old man presents with complaints of severe pain in his left wrist that he says is episodic and has increased in frequency over the past year. He says that he cannot move the wrist when this happens. His father had similar problems before he died of kidney problems due to diabetes. The patient is also diabetic and is taking insulin. His physical examination is normal, except for the limitation of motion of the left wrist. A CBC is normal, and serum chemistry findings are as follows:

Sodium: 139 mEq/L
Potassium: 3.9 mEq/L
Chloride: 98 mEq/L
Calcium: 9.2 mEq/L
Uric acid: 4 mg/dL

Synovial fluid analysis reveals a white blood cell count of 32,000/μL with 60% neutrophils. Rhomboid-shaped, positive, birefringent crystals are seen under polarized light. Which of the following is the most likely diagnosis in this patient?

(A) Charcot arthropathy
(B) Degenerative joint disease
(C) Gout
(D) Pseudogout
(E) Rheumatoid arthritis
(F) Septic arthritis

A

Respuesta: D
Carlos: D

45
Q

A 52-year-old woman from Southeast Asia comes to medical attention because of slowly growing nodular lesions on her nose and auricles. Examination also reveals bilateral hypoesthesia in the upper extremities along the ulnar nerve distribution. Biopsies of the skin lesions demonstrate a florid granulomatous reaction with numerous acid-fast bacilli (AFB) within multinucleated giant cells. Cultures on blood agar and special media, however, yield no growth. Which of the following is the most likely diagnosis?

(A) Lepromatous leprosy
(B) Lupus vulgaris
(C) Mycobacterium avium-intracellulare (MAI)
(D) Sarcoidosis
(E) Tuberculoid leprosy

A

Respuesta: A
Vero: C

46
Q

A 42-year-old man presents with a chief complaint of severe,
sharp chest pain that started suddenly while lifting heavy objects.
The pain began in a midsternal location, then radiated to both
shoulders as well as to his back. It has been constant for 18 hours
but started to get worse during the past 2 hours. On physical
examination, the patient is in severe distress, with a temperature of
36.9 C (98.5 F), blood pressure of 160/90 mm Hg, pulse of 92/ min,
and respirations of 18/min. Heart sounds are normal without rubs or
murmur. An ECG reveals a normal tracing. Which of the following
is the most appropriate next step in diagnosis?
(A) Chest x-ray film
(B) CT scan of chest
(C) MRI
(D) Ventilation-perfusion (V/Q) scan
(E) Angiogram

47
Q

A 55-year-old, homeless, alcoholic man who has recently been binging complains of 2 weeks of fever, malaise, productive cough, and pain on deep inspiration. He has smoked two packs of cigarettes per day for the past 30 years. A chest x-ray film reveals an infiltrate of the superior portion of the right lower lobe, with a cavity containing an air fluid level. A biopsy is likely to show which of the following?

(A) Acid-fast bacilli and caseating granulomas
(B) Anaplastic squamous cells with numerous mitotic figures
(C) Fibrosis and needle-like ferruginous bodies
(D) Gram-positive, lancet-shaped diplococci in short chains
(E) Mixture of anaerobic organisms

A

Respuesta: E

The correct answer is E. This patient’s signs and symptoms indicate aspiration pneumonia with resultant lung abscess. Lung abscesses in an unconscious or obtunded patient result from aspiration of infected material from the upper airway. The causative organisms are usually mixed anaerobes from oral flora. This patient’s infiltrate is in the superior portion of the right lower lobe, a common site of aspiration pneumonia. Sputum is copious in lung abscesses and often malodorous. Patients have risk factors for aspiration (drugs, CNS disease, general anesthesia, coma, or excessive sedation), and many patients have periodontal infection.

Acid-fast bacilli and caseating granulomas (choice A) refer to pulmonary tuberculosis. Although it may be seen in homeless alcoholics, tuberculosis more often presents with a protracted, rather than an acute, illness. Additionally, cavity formation classically occurs in the upper lung lobes.

Considering this patient’s long history of tobacco use, a possible diagnosis is squamous cell carcinoma (choice B), which may lead to a mass with an air-fluid level that resembles a lung abscess radiographically. Additionally, some lung cancers can present with a postobstructive pneumonia with resultant lung abscess. However, lung cancer usually presents with symptoms over longer periods of time.

Fibrosis and needle-like ferruginous bodies (choice C) are the classic histology for asbestosis. Asbestosis results from long-term exposure to asbestos fibers and presents with progressive exertional dyspnea and fibrosis. Symptoms are not as acute as those found in this patient, and a history of asbestos or occupational exposure is needed for the diagnosis.

Pneumococcal pneumonia (choice D) is the most common community-acquired pneumonia. It is a lobar pneumonia that does not cavitate or cause lung abscesses. Additionally, presentation of fever, chest pain, dyspnea, and cough occurs more acutely than the 2-week presentation seen in this patient.

48
Q

An 83-year-old woman presents with a 1-year history of increasing forgetfulness and inattentiveness. She has had episodes of confusion, usually occurring at night when she wanders around in her house, disoriented to time and place. According to a family member, she has recently developed paranoid ideation. On a Mini- Mental Status examination, she is unable to recall one of three words, but she is able to follow a three-stage command. There is no history of alcohol abuse, major physical illness, or current pharmacologic therapy. Physical examination is unrevealing, and blood and thyroid function tests are within normal limits. Which of the following is the most appropriate next step in diagnosis?

(A) Electroencephalographic studies
(B) MRI of the brain
(C) Cerebral angiographic studies
(D) Lumbar puncture for CSF examination
(E) Brain biopsy

A

Respuesta: B

The correct answer is B. Even when the clinical history and neurologic evaluation are consistent with Alzheimer disease, as in this case, MRI studies of the brain may rule out other conditions that may mimic this common dementing disorder. Marked ventricular dilatation without significant cortical atrophy, for example, would suggest normal pressure hydrocephalus, which may respond to CSF shunting. A subdural hematoma may manifest with dementia. An entirely normal-appearing brain may suggest nonorganic causes of dementia, for example.

Electroencephalographic studies (choice A) are of limited value in the evaluation of dementing illnesses, except in the case of Creutzfeldt-Jacob disease.

Cerebral angiographic studies (choice C) are useful in evaluating the morphology of the cerebral vasculature to detect the location and degree of atherosclerotic stenosis, as well as the presence of aneurysms or vascular malformations.

Lumbar puncture for CSF examination (choice D) is uninformative in most cases of dementia. Despite extensive research, a diagnostically useful CSF marker of Alzheimer disease has yet to be found.

Brain biopsy (choice E) is reserved for those rare cases in which clinical and radiologic investigations have failed to disclose any apparent cause of dementia, and there is evidence of a mass lesion in imaging studies.

49
Q

A 45-year-old man presents with respiratory difficulty and right chest pain. His temperature is 37.7 C (99.9 F) and respirations are 24/min and shallow. Chest examination reveals decreased tactile fremitus over the right hemithorax, with an extensive area of dullness to percussion and marked diminution of breath sounds on auscultation. Just above this area, egophony is appreciated.

(A) Acute respiratory distress syndrome
(B) Asthma
(C) Bronchiectasis
(D) Carcinoma of the lungs
(E) Chronic aspiration of gastric contents
(F) Chronic obstructive pulmonary disease (COPD)
(G) Cystic fibrosis
(H) Pleural effusion
(I) Pneumocystis carinii pneumonia
(J) Pulmonary hypertension
(K) Pulmonary thromboembolism
(L) Sarcoidosis
(M) Spontaneous pneumothorax
(N) Tuberculosis

50
Q

A 38-year-old black woman presents with the insidious onset of shortness of breath, chest pain, and fatigue. Physical examination reveals enlarged cervical lymph nodes and scattered brown-red papules on the skin. A chest x-ray film shows bilateral pulmonary infiltrates and enlarged hilar lymph nodes. Biopsies of skin lesions and lymph nodes show non-necrotizing granulomas. Special stains for fungi and mycobacteria are negative.

(A) Acute respiratory distress syndrome
(B) Asthma
(C) Bronchiectasis
(D) Carcinoma of the lungs
(E) Chronic aspiration of gastric contents
(F) Chronic obstructive pulmonary disease (COPD)
(G) Cystic fibrosis
(H) Pleural effusion
(I) Pneumocystis carinii pneumonia
(J) Pulmonary hypertension
(K) Pulmonary thromboembolism
(L) Sarcoidosis
(M) Spontaneous pneumothorax
(N) Tuberculosis